Help integrating the contour integral $\oint_C \frac{e^{\frac{1}{z}}}{z(1-qz)}dz$ around the unit circle

complex-analysiscontour-integrationexponential functionintegration

Consider the integral $$\oint_C \frac{e^{\frac{1}{z}}}{z(1-qz)}dz$$

where C is the anti-clockwise oriented unit circle and q is a complex constant.

I have no clue how to integrate this. I tried using the residue theorem but it just doesn't come out correctly. Any help would be appreciated. The reason I am stuck is because it seems like there are an infinite amount of poles at z =0, so am very confused

Best Answer

Following the recommendation of @honey kumar you can expend the integrand into the series. The integrand is analytic in all points of the unit circle, except for $z=0$. So, you can deform the contour into a circle of smaller radius $r\to0$ around $z=0$, and then to choose all terms of the integrand expansion into series containing $\frac{1}{z}$, because only these terms will contribute, according to the residual theorem.

$\frac{e^{\frac{1}{z}}}{z(1-qz)}=\frac{1}{z}\left(1+\frac{1}{z1!}+\frac{1}{z^22!}+\frac{1}{z^33!}+...\right)\left(1+qz+q^2z^2+q^3z^3+...\right)$

You need to take only simple poles from this expression. It gives:

$I(q)=\oint_C \frac{e^{\frac{1}{z}}}{z(1-qz)}dz=\oint_C\frac{dz}{z}\left(1+\frac{qz}{z1!}+\frac{q^2z^2}{z^22!}+\frac{q^3z^3}{z^33!}+...\right)=\oint_C\frac{dz}{z}e^q=2\pi{i}e^q$

An important point:

thanks to @A rural reader who points out that $I(q)=2\pi{i}e^q$ at $|q|<1$ and that at $|q|>1$ the integral is zero.

Indeed, when we squeeze the contour ($|z|=r\to0$) and if $|q|>1$ the integration path will have to cross the point $z=\frac{1}{q}$ first, and only after we can enjoy the expansion of $\frac{1}{1-qz}=1=qz+q^2z^2+...$. When crossing the point $z=\frac{1}{q}$ we will get an additional residual in this point ($=-e^q$) which will cancel the residual at $z=0$.

We can also show that $I(|q|>1)=0$ directly:

$I(q)=\oint_C \frac{e^{\frac{1}{z}}}{z(1-qz)}dz=-\oint_C \frac{e^{\frac{1}{z}}}{qz^2(1-\frac{1}{qz})}dz=$$=-\oint_C\frac{dz}{qz^2}\left(1+\frac{1}{z1!}+\frac{1}{z^22!}+\frac{1}{z^33!}+...\right)\left(1+\frac{1}{qz}+\frac{1}{q^2z^2}+...\right)=0$

because there is no simple poles at $z=0$.

Related Question